Aides sur les series Urgent avant Samedi

Aide sur les questions d'analyses.
Julien_
Membre
Messages : 7
Inscription : 19 mars 2014, 23:14

Aides sur les series Urgent avant Samedi

Message par Julien_ » 21 avril 2016, 15:08

Bonjour,

Ci-joint les 2 exercices que je n'arrives pas à résoudre, ci vous pouvez me donnez un petit coup de main svp ;)

rsz_320160421_144032_hdr.jpg
rsz_320160421_144032_hdr.jpg (103.01 Kio) Consulté 3925 fois

Merci d'avance :) :) :)

Avatar de l’utilisateur
Job
Propriétaire du forum
Messages : 2584
Inscription : 28 juin 2013, 15:07
Contact :

Re: Aides sur les series Urgent avant Samedi

Message par Job » 21 avril 2016, 17:53

Bonjour

Pour les séries de Bertrand, une démonstration sur le net :
http://uel.unisciel.fr/mathematiques/se ... re_19.html

Julien_
Membre
Messages : 7
Inscription : 19 mars 2014, 23:14

Re: Aides sur les series Urgent avant Samedi

Message par Julien_ » 22 avril 2016, 13:15

Merci Job :) et pour le deuxième exercice (exo 7) svp .
P.S : Je veux bien aussi la correction de l'exo 6 pour être bien sûr

Avatar de l’utilisateur
Job
Propriétaire du forum
Messages : 2584
Inscription : 28 juin 2013, 15:07
Contact :

Re: Aides sur les series Urgent avant Samedi

Message par Job » 22 avril 2016, 16:28

Bonjour

Exercice 7
a) La fonction inverse est décroissante sur ${\mathbb R}^{+,*}$ donc
$\forall k\geq 1,\ \int_k^{k+1} \frac{dt}{t^{\alpha}}\leq \frac{1}{k^{\alpha}}$
En sommant $\sum_{k=1}^{\infty} \frac{1}{k^{\alpha}}\geq \sum_{k=1}^{\infty}\int_k^{k+1} \frac{dt}{t^{\alpha}}=\int_1^{\infty}\frac{dt}{t^{\alpha}}=[\frac{t^{-\alpha +1}}{-\alpha +1}]_1^{\infty}=\frac{1}{\alpha -1}$

$\forall k\geq 2,\ \frac{1}{k^{\alpha}}\leq \int_{k-1}^k\frac{dt}{t^{\alpha}}$
$\sum_{k=1}^{\infty}\frac{1}{k^{\alpha}}\leq 1+\sum_{k=2}^{\infty}\int_{k-1}^{\infty} \frac{dt}{t^{\alpha}}=1+\int_1^{\infty} \frac{dt}{t^{\alpha}}=1+\frac{1}{\alpha -1}$

On a donc : $\frac{1}{\alpha -1}\leq \zeta (\alpha)\leq 1+\frac{1}{\alpha -1}$
$1\leq (\alpha -1)\zeta (\alpha)\leq \alpha$ donc $\lim_{\alpha \to 1} (\alpha -1)\zeta (\alpha)=1$ et par conséquent $\zeta (\alpha) \sim \frac{1}{\alpha -1}$


b) On repart des mêmes inégalités en se limitant à $n$
$\sum_{k=1}^n \frac{1}{k^{\alpha}}\geq \sum_1^{n+1}\frac{dt}{t^{\alpha}}=[\frac{t^{-\alpha+1}}{-\alpha +1}]_1^{n+1}=\frac{1}{1-\alpha}((n+1)^{\alpha}-1)$

$\sum_{k=1}^n \frac{1}{k^{\alpha}}\leq 1+\int_1^n \frac{dt}{t^{\alpha}}=1+\frac{1}{1-\alpha}(n^{1-\alpha}-1)$

$\frac{1}{1-\alpha}((n+1)^{1-\alpha}-1\leq S_n\leq \frac{1}{1-\alpha}(n^{1-\alpha}-1)+1$
$\frac{(n+1)^{1-\alpha}}{n^{1-\alpha}}-\frac{1}{n^{1-\alpha}}\leq \frac{1-\alpha}{n^{1-\alpha}} S_n \leq 1+\frac{-1+1-\alpha}{n^{1-\alpha}}$

$\lim_{n\to +\infty}\frac{1}{n^{1-\alpha}}=\lim_{n\to \infty}\frac{\alpha}{n^{1-\alpha}}=0$

$\ln (1+\frac{1}{n})^{1-\alpha}=(1-\alpha) \ln (1+\frac{1}{n})\sim (1-\alpha)\times \frac{1}{n}$ donc $\lim_{n\to +\infty}\ln (1+\frac{1}{n})^{1-\alpha}=0$ et par conséquent $\lim_{n\to +\infty} (1+\frac{1}{n})^{1-\alpha}=1$

Donc $\lim_{n\to +\infty} \frac{1-\alpha}{n^{1-\alpha}} S_n=1$ et $S_n\sim \frac{n^{1-\alpha}}{1-\alpha}$

Julien_
Membre
Messages : 7
Inscription : 19 mars 2014, 23:14

Re: Aides sur les series Urgent avant Samedi

Message par Julien_ » 22 avril 2016, 17:26

Merci beaucoup Job , au final c'est ok :-)

Répondre